Decision problem that can be verified but not run in n^2 timeA Problem on Time Complexity of AlgorithmsCan we...

Current across a wire with zero potential difference

Citing paywalled articles accessed via illegal web sharing

Is there a lava-breathing lizard creature (that could be worshipped by a cult) in 5e?

Do "fields" always combine by addition?

Why is it that Bernie Sanders is always called a "socialist"?

How would an AI self awareness kill switch work?

Is a new boolean field better than null reference when a value can be meaningfully absent?

Why avoid shared user accounts?

After checking in online, how do I know whether I need to go show my passport at airport check-in?

Separate environment for personal and development use under macOS

Is there a verb that means to inject with poison?

What is the difference between "...", '...', $'...', and $"..." quotes?

Why was Lupin comfortable with saying Voldemort's name?

Is there a defined priority for pattern matching?

Is the child responsible for the Parent PLUS Loan when the parent has passed away?

How to not let the Identify spell spoil everything?

Non-Cancer terminal illness that can affect young (age 10-13) girls?

Can I announce prefix 161.117.25.0/24 even though I don't have all of /24 IPs

How does Leonard in "Memento" remember reading and writing?

Existence of Riemann surface, holomorphic maps

Why do neural networks need so many training examples to perform?

Explanation of a regular pattern only occuring for prime numbers

Why didn't Tom Riddle take the presence of Fawkes and the Sorting Hat as more of a threat?

Can you tell from a blurry photo if focus was too close or too far?



Decision problem that can be verified but not run in n^2 time


A Problem on Time Complexity of AlgorithmsCan we construct problems that can be solved in $Theta(n^c)$ time, and tested in $O(n)$ timeProblems that provably require quadratic timeIs there a decision algorithm with time complexity of Ө(n²)?Lower bounds and $P$ vs $NP$Is P the set of all algorithms whose run-time is $Oleft( n^{ O left( 1 right) }right)$?Does there exist a Turing-machine that runs in time $o(nlog n)$, but not $O(n)$?reducing $CLIQUE$ from decision to search problemHow is the Time Complexity of a Function Problem different than its corresponding Decision Problem?Show that NP is not equal to SPACE(n)













2












$begingroup$


A much much weaker idea similar to the P=NP question, is there a decision problem that can be verified in $O(n^2)$ time, but it can be proven that there is no algorithm that decides it $O(n^2)$ time?










share|cite|improve this question











$endgroup$












  • $begingroup$
    In what model of computation?
    $endgroup$
    – Curtis F
    3 hours ago










  • $begingroup$
    @CurtisF if it matters then please write that up as an answer. I was under the impression that that sort of thing didn't matter within reason, or else how can one ask "does P=NP"?
    $endgroup$
    – while1fork
    3 hours ago






  • 3




    $begingroup$
    @while1fork. It does matter if you're giving precise timing estimates, like $O(n^2)$.
    $endgroup$
    – Rick Decker
    3 hours ago


















2












$begingroup$


A much much weaker idea similar to the P=NP question, is there a decision problem that can be verified in $O(n^2)$ time, but it can be proven that there is no algorithm that decides it $O(n^2)$ time?










share|cite|improve this question











$endgroup$












  • $begingroup$
    In what model of computation?
    $endgroup$
    – Curtis F
    3 hours ago










  • $begingroup$
    @CurtisF if it matters then please write that up as an answer. I was under the impression that that sort of thing didn't matter within reason, or else how can one ask "does P=NP"?
    $endgroup$
    – while1fork
    3 hours ago






  • 3




    $begingroup$
    @while1fork. It does matter if you're giving precise timing estimates, like $O(n^2)$.
    $endgroup$
    – Rick Decker
    3 hours ago
















2












2








2





$begingroup$


A much much weaker idea similar to the P=NP question, is there a decision problem that can be verified in $O(n^2)$ time, but it can be proven that there is no algorithm that decides it $O(n^2)$ time?










share|cite|improve this question











$endgroup$




A much much weaker idea similar to the P=NP question, is there a decision problem that can be verified in $O(n^2)$ time, but it can be proven that there is no algorithm that decides it $O(n^2)$ time?







time-complexity






share|cite|improve this question















share|cite|improve this question













share|cite|improve this question




share|cite|improve this question








edited 4 hours ago







while1fork

















asked 4 hours ago









while1forkwhile1fork

183




183












  • $begingroup$
    In what model of computation?
    $endgroup$
    – Curtis F
    3 hours ago










  • $begingroup$
    @CurtisF if it matters then please write that up as an answer. I was under the impression that that sort of thing didn't matter within reason, or else how can one ask "does P=NP"?
    $endgroup$
    – while1fork
    3 hours ago






  • 3




    $begingroup$
    @while1fork. It does matter if you're giving precise timing estimates, like $O(n^2)$.
    $endgroup$
    – Rick Decker
    3 hours ago




















  • $begingroup$
    In what model of computation?
    $endgroup$
    – Curtis F
    3 hours ago










  • $begingroup$
    @CurtisF if it matters then please write that up as an answer. I was under the impression that that sort of thing didn't matter within reason, or else how can one ask "does P=NP"?
    $endgroup$
    – while1fork
    3 hours ago






  • 3




    $begingroup$
    @while1fork. It does matter if you're giving precise timing estimates, like $O(n^2)$.
    $endgroup$
    – Rick Decker
    3 hours ago


















$begingroup$
In what model of computation?
$endgroup$
– Curtis F
3 hours ago




$begingroup$
In what model of computation?
$endgroup$
– Curtis F
3 hours ago












$begingroup$
@CurtisF if it matters then please write that up as an answer. I was under the impression that that sort of thing didn't matter within reason, or else how can one ask "does P=NP"?
$endgroup$
– while1fork
3 hours ago




$begingroup$
@CurtisF if it matters then please write that up as an answer. I was under the impression that that sort of thing didn't matter within reason, or else how can one ask "does P=NP"?
$endgroup$
– while1fork
3 hours ago




3




3




$begingroup$
@while1fork. It does matter if you're giving precise timing estimates, like $O(n^2)$.
$endgroup$
– Rick Decker
3 hours ago






$begingroup$
@while1fork. It does matter if you're giving precise timing estimates, like $O(n^2)$.
$endgroup$
– Rick Decker
3 hours ago












1 Answer
1






active

oldest

votes


















3












$begingroup$

The answer depends a lot on the model of computation.



On one extreme: in the decision tree model, many lower bounds can be proven. For instance, consider the 3SUM problem. You can verify an alleged solution to the 3SUM problem in $O(n)$ time, but it's conjectured that no algorithm can solve the 3SUM problem in $O(n^{2-epsilon})$ time for any $epsilon>0$. One can prove this holds in some version of the algebraic decision tree model; one can prove that any algorithm to solve it in this model must take $Omega(n^2)$ time. This provides a gap of $O(n^2)$ vs $O(n)$ for solving vs verifying solutions, if the verifier is limited to the algebraic decision tree model. This is a pretty limited result; in this case, the model means that we're restricting attention to algorithms of a particular form. So, it doesn't rule out the possibility of some other algorithm (that does something weird) being faster.



On the other extreme: if we allow arbitrary (non-uniform) boolean circuits, then there is no explicitly stated function $f$ on $n$ bits where we can currently prove that every circuit for computing $f$ needs $ge 3.1n$ gates. In other words, this is saying that we have no clue how to prove lower bounds for circuits. Roughly speaking, we have no known result of a problem where we can prove that solving it requires circuits of size $omega(n)$. See, e.g., https://cstheory.stackexchange.com/a/8005/5038 and https://cstheory.stackexchange.com/q/21400/5038. So, given our current state of knowledge, we have no hope of proving a result like that if the model of computation is unrestricted boolean circuits.






share|cite|improve this answer









$endgroup$













    Your Answer





    StackExchange.ifUsing("editor", function () {
    return StackExchange.using("mathjaxEditing", function () {
    StackExchange.MarkdownEditor.creationCallbacks.add(function (editor, postfix) {
    StackExchange.mathjaxEditing.prepareWmdForMathJax(editor, postfix, [["$", "$"], ["\\(","\\)"]]);
    });
    });
    }, "mathjax-editing");

    StackExchange.ready(function() {
    var channelOptions = {
    tags: "".split(" "),
    id: "419"
    };
    initTagRenderer("".split(" "), "".split(" "), channelOptions);

    StackExchange.using("externalEditor", function() {
    // Have to fire editor after snippets, if snippets enabled
    if (StackExchange.settings.snippets.snippetsEnabled) {
    StackExchange.using("snippets", function() {
    createEditor();
    });
    }
    else {
    createEditor();
    }
    });

    function createEditor() {
    StackExchange.prepareEditor({
    heartbeatType: 'answer',
    autoActivateHeartbeat: false,
    convertImagesToLinks: false,
    noModals: true,
    showLowRepImageUploadWarning: true,
    reputationToPostImages: null,
    bindNavPrevention: true,
    postfix: "",
    imageUploader: {
    brandingHtml: "Powered by u003ca class="icon-imgur-white" href="https://imgur.com/"u003eu003c/au003e",
    contentPolicyHtml: "User contributions licensed under u003ca href="https://creativecommons.org/licenses/by-sa/3.0/"u003ecc by-sa 3.0 with attribution requiredu003c/au003e u003ca href="https://stackoverflow.com/legal/content-policy"u003e(content policy)u003c/au003e",
    allowUrls: true
    },
    onDemand: true,
    discardSelector: ".discard-answer"
    ,immediatelyShowMarkdownHelp:true
    });


    }
    });














    draft saved

    draft discarded


















    StackExchange.ready(
    function () {
    StackExchange.openid.initPostLogin('.new-post-login', 'https%3a%2f%2fcs.stackexchange.com%2fquestions%2f104894%2fdecision-problem-that-can-be-verified-but-not-run-in-n2-time%23new-answer', 'question_page');
    }
    );

    Post as a guest















    Required, but never shown

























    1 Answer
    1






    active

    oldest

    votes








    1 Answer
    1






    active

    oldest

    votes









    active

    oldest

    votes






    active

    oldest

    votes









    3












    $begingroup$

    The answer depends a lot on the model of computation.



    On one extreme: in the decision tree model, many lower bounds can be proven. For instance, consider the 3SUM problem. You can verify an alleged solution to the 3SUM problem in $O(n)$ time, but it's conjectured that no algorithm can solve the 3SUM problem in $O(n^{2-epsilon})$ time for any $epsilon>0$. One can prove this holds in some version of the algebraic decision tree model; one can prove that any algorithm to solve it in this model must take $Omega(n^2)$ time. This provides a gap of $O(n^2)$ vs $O(n)$ for solving vs verifying solutions, if the verifier is limited to the algebraic decision tree model. This is a pretty limited result; in this case, the model means that we're restricting attention to algorithms of a particular form. So, it doesn't rule out the possibility of some other algorithm (that does something weird) being faster.



    On the other extreme: if we allow arbitrary (non-uniform) boolean circuits, then there is no explicitly stated function $f$ on $n$ bits where we can currently prove that every circuit for computing $f$ needs $ge 3.1n$ gates. In other words, this is saying that we have no clue how to prove lower bounds for circuits. Roughly speaking, we have no known result of a problem where we can prove that solving it requires circuits of size $omega(n)$. See, e.g., https://cstheory.stackexchange.com/a/8005/5038 and https://cstheory.stackexchange.com/q/21400/5038. So, given our current state of knowledge, we have no hope of proving a result like that if the model of computation is unrestricted boolean circuits.






    share|cite|improve this answer









    $endgroup$


















      3












      $begingroup$

      The answer depends a lot on the model of computation.



      On one extreme: in the decision tree model, many lower bounds can be proven. For instance, consider the 3SUM problem. You can verify an alleged solution to the 3SUM problem in $O(n)$ time, but it's conjectured that no algorithm can solve the 3SUM problem in $O(n^{2-epsilon})$ time for any $epsilon>0$. One can prove this holds in some version of the algebraic decision tree model; one can prove that any algorithm to solve it in this model must take $Omega(n^2)$ time. This provides a gap of $O(n^2)$ vs $O(n)$ for solving vs verifying solutions, if the verifier is limited to the algebraic decision tree model. This is a pretty limited result; in this case, the model means that we're restricting attention to algorithms of a particular form. So, it doesn't rule out the possibility of some other algorithm (that does something weird) being faster.



      On the other extreme: if we allow arbitrary (non-uniform) boolean circuits, then there is no explicitly stated function $f$ on $n$ bits where we can currently prove that every circuit for computing $f$ needs $ge 3.1n$ gates. In other words, this is saying that we have no clue how to prove lower bounds for circuits. Roughly speaking, we have no known result of a problem where we can prove that solving it requires circuits of size $omega(n)$. See, e.g., https://cstheory.stackexchange.com/a/8005/5038 and https://cstheory.stackexchange.com/q/21400/5038. So, given our current state of knowledge, we have no hope of proving a result like that if the model of computation is unrestricted boolean circuits.






      share|cite|improve this answer









      $endgroup$
















        3












        3








        3





        $begingroup$

        The answer depends a lot on the model of computation.



        On one extreme: in the decision tree model, many lower bounds can be proven. For instance, consider the 3SUM problem. You can verify an alleged solution to the 3SUM problem in $O(n)$ time, but it's conjectured that no algorithm can solve the 3SUM problem in $O(n^{2-epsilon})$ time for any $epsilon>0$. One can prove this holds in some version of the algebraic decision tree model; one can prove that any algorithm to solve it in this model must take $Omega(n^2)$ time. This provides a gap of $O(n^2)$ vs $O(n)$ for solving vs verifying solutions, if the verifier is limited to the algebraic decision tree model. This is a pretty limited result; in this case, the model means that we're restricting attention to algorithms of a particular form. So, it doesn't rule out the possibility of some other algorithm (that does something weird) being faster.



        On the other extreme: if we allow arbitrary (non-uniform) boolean circuits, then there is no explicitly stated function $f$ on $n$ bits where we can currently prove that every circuit for computing $f$ needs $ge 3.1n$ gates. In other words, this is saying that we have no clue how to prove lower bounds for circuits. Roughly speaking, we have no known result of a problem where we can prove that solving it requires circuits of size $omega(n)$. See, e.g., https://cstheory.stackexchange.com/a/8005/5038 and https://cstheory.stackexchange.com/q/21400/5038. So, given our current state of knowledge, we have no hope of proving a result like that if the model of computation is unrestricted boolean circuits.






        share|cite|improve this answer









        $endgroup$



        The answer depends a lot on the model of computation.



        On one extreme: in the decision tree model, many lower bounds can be proven. For instance, consider the 3SUM problem. You can verify an alleged solution to the 3SUM problem in $O(n)$ time, but it's conjectured that no algorithm can solve the 3SUM problem in $O(n^{2-epsilon})$ time for any $epsilon>0$. One can prove this holds in some version of the algebraic decision tree model; one can prove that any algorithm to solve it in this model must take $Omega(n^2)$ time. This provides a gap of $O(n^2)$ vs $O(n)$ for solving vs verifying solutions, if the verifier is limited to the algebraic decision tree model. This is a pretty limited result; in this case, the model means that we're restricting attention to algorithms of a particular form. So, it doesn't rule out the possibility of some other algorithm (that does something weird) being faster.



        On the other extreme: if we allow arbitrary (non-uniform) boolean circuits, then there is no explicitly stated function $f$ on $n$ bits where we can currently prove that every circuit for computing $f$ needs $ge 3.1n$ gates. In other words, this is saying that we have no clue how to prove lower bounds for circuits. Roughly speaking, we have no known result of a problem where we can prove that solving it requires circuits of size $omega(n)$. See, e.g., https://cstheory.stackexchange.com/a/8005/5038 and https://cstheory.stackexchange.com/q/21400/5038. So, given our current state of knowledge, we have no hope of proving a result like that if the model of computation is unrestricted boolean circuits.







        share|cite|improve this answer












        share|cite|improve this answer



        share|cite|improve this answer










        answered 2 hours ago









        D.W.D.W.

        99.8k12121286




        99.8k12121286






























            draft saved

            draft discarded




















































            Thanks for contributing an answer to Computer Science Stack Exchange!


            • Please be sure to answer the question. Provide details and share your research!

            But avoid



            • Asking for help, clarification, or responding to other answers.

            • Making statements based on opinion; back them up with references or personal experience.


            Use MathJax to format equations. MathJax reference.


            To learn more, see our tips on writing great answers.




            draft saved


            draft discarded














            StackExchange.ready(
            function () {
            StackExchange.openid.initPostLogin('.new-post-login', 'https%3a%2f%2fcs.stackexchange.com%2fquestions%2f104894%2fdecision-problem-that-can-be-verified-but-not-run-in-n2-time%23new-answer', 'question_page');
            }
            );

            Post as a guest















            Required, but never shown





















































            Required, but never shown














            Required, but never shown












            Required, but never shown







            Required, but never shown

































            Required, but never shown














            Required, but never shown












            Required, but never shown







            Required, but never shown







            Popular posts from this blog

            Paper upload error, “Upload failed: The top margin is 0.715 in on page 3, which is below the required...

            Emraan Hashmi Filmografia | Linki zewnętrzne | Menu nawigacyjneGulshan GroverGulshan...

            How can I write this formula?newline and italics added with leqWhy does widehat behave differently if I...